Klassische Elektrodynamik der kleinsten Wirkung ohne Potentiale

Ist es möglich, die klassische Elektrodynamik (im Sinne der Ableitung der Maxwell-Gleichungen) aus einem Prinzip der kleinsten Wirkung ohne die Verwendung von Potentialen zu formulieren? Das heißt, gibt es eine Lagrange-Funktion, die nur von den elektrischen und magnetischen Feldern abhängt und die Maxwell-Gleichungen als Euler-Lagrange-Gleichungen haben wird?

Ich kann wirklich keinen Zusammenhang zwischen der Herleitung von Maxwell-Gl. vom Prinzip der kleinsten Wirkung und dem Verzicht auf Potentiale, können wir an jeder Stelle Potentiale ein- oder ausschließen, denn in der klassischen Physik (nicht Quantenphysik) ist dies nur eine Frage der Definitionen. In Bezug auf die Ableitung der kleinsten Aktion müssen wir, um sie zu verwenden, zunächst eine Lagrange-Funktion haben. Wenn Sie davon ausgehen, dass sie gegeben ist, können Sie natürlich alle Maxwell-Gleichungen ableiten. Wie auch immer, eine interessante Sache, die ich weiß, ist, dass, wenn Sie nur annehmen, dass wir eine Kraft haben, die sich wie die Schwerkraft verhält (zentral und proportional zu den Abständen im Quadrat
Als ich die Antworten durchgehe, sagt mir Occams Rasiermesser, dass dies nicht der richtige Weg ist.

Antworten (4)

1) Nun, auf der klassischen Ebene können wir, wenn wir Hilfsvariablen einführen dürfen, immer trivial einen Satz von Bewegungsgleichungen codieren

(1) e Ö m ich = 0 , ich { 1 , , n } ,
auf Brute-Force-Weise mit Hilfe von Lagrange-Multiplikatoren
(2) λ ich , ich { 1 , , n } ,
so dass die Lagrange-Dichte einfach lautet
(3) L   =   ich = 1 n λ ich   e Ö m ich .

Dies ist aus vielen Gründen keine sehr zufriedenstellende Lösung. (Besonders wenn wir beginnen, über quantenmechanische Aspekte nachzudenken. OP fragt jedoch nur nach klassischer Physik.) Nichtsdestotrotz veranschaulichen die obigen trivialen Umschreibungen (3), wie schwierig es ist, No-Go-Theoreme mit stichhaltigen Argumenten zu formulieren und zu beweisen.

2) Um fortzufahren, müssen wir zusätzliche Bedingungen an die Form des Aktionsprinzips stellen. Erstens, weil es uns verboten ist, Eichpotentiale einzuführen EIN μ Als fundamentale Variablen (die wir im Wirkungsprinzip variieren können) nehmen wir an, dass die fundamentalen EM-Variablen im Vakuum durch die gegeben sein sollten E und B Feld. Schon in reinem EM ist es unmöglich das zu bekommen 1 + 1 + 3 + 3 = 8 Maxwell-Gleichungen. (in Differentialform) als Euler-Lagrange-Gleichungen. indem man nur die variiert 3 + 3 = 6 Feldvariablen E und B . Wir müssten also auf die eine oder andere Weise zusätzliche Feldvariablen einführen.

3a) Es wird nicht besser, wenn wir versuchen, EM an Materie zu koppeln. Indem wir Ecken der Theorie entkoppeln, sollten wir in der Lage sein, wohlbekannte Spezialfälle wiederzufinden. Zum Beispiel im Fall von EM, das an geladene Punktteilchen gekoppelt ist, sagen wir in einer nicht-relativistischen Grenze, wo es kein EM-Feld gibt, sollte sich die Lagrange-Funktion einer einzelnen Punktladung auf die wohlbekannte Form reduzieren

(4) L   =   1 2 m v 2

eines freien Teilchens. Eine Diskussion von Gl. (4) findet sich zB in diesem Phys.SE Beitrag. Hier nehmen wir an, dass Gl. (4) gilt im Folgenden.

3b) Die nächste Frage ist, was in der Elektrostatik passiert

(5) m v ˙   =   q E ?

Die Antwort ist bekannt

(6) L   =   1 2 m v 2 v

mit potentieller Energie

(7) v   =   q ϕ ,

wo ϕ ist das skalare elektrische Potential. Da ist es uns jedoch untersagt, das Potential einzubringen ϕ als fundamentale Variable müssen wir sie interpretieren

(8) ϕ ( r )   :=   r d r ' E ( r ' )

als Funktion des elektrischen Feldes E , das wiederum als Grundfeld angenommen wird. Beachten Sie, dass Gl. (6)-(8) entsprechen einer nichtlokalen Aktion.

3c) Die einfache Verallgemeinerung (von der Punktmechanik zur Feldtheorie) von Gl. (7) ist eine Potentialdichte

(9) v   =   ρ ϕ ,

wo ρ ist eine elektrische Ladungsdichte. Leser, die mit dem üblichen Aktionsprinzip für Maxwells Gleichungen vertraut sind. werden erkennen, dass wir sehr nahe daran sind zu argumentieren, dass der Wechselwirkungsterm zwischen reinem EM und Materie die Form haben muss

(10) L ich n t   =   J μ EIN μ ,

auch wenn wir noch nicht darüber gesprochen haben, was den Standard-Lagrange ersetzen soll

(11) L E M   =   1 4 F μ v F μ v

für reine EM.

3d) Bleiben wir in der Elektrostatik und denken wir über unsere Aussichten nach, das Gaußsche Gesetz in Differentialform abzuleiten

(12) E   =   ρ .

Offensichtlich die rechte. der einzelnen Gl. (12) sollte durch Variation der Potentialdichte (9) bzgl. einer der drei E Felder, aber welches? Die Zählung stimmt nicht. Und weil Gl. (9) ist nicht lokal, wir bekommen auf jeden Fall eine integrierte Version davon ρ eher, als ρ selbst, das auf der rechten Seite erscheint. von Gl. (12), die wir reproduzieren wollten.

3e) Zusammenfassend erscheint es aussichtslos, eine EM-Theorie (mit E und B als fundamentale Variablen) zur Materie und reproduzieren klassische Standardgleichungen. der Bewegung.

4) Das Standardmittel ist die Einführung 4 (global definierte) Eichpotentiale EIN μ als fundamentale Variablen. Das macht 1 + 3 = 4 quellenlose Maxwell-Gl. trivial, und die restlichen 1 + 3 = 4 Maxwell-Gleichungen. mit Quellen können durch Variieren wrt abgeleitet werden. der 4 grundlegende Variablen EIN μ .

Zum Beispiel die standardmäßige (spezielle relativistische) Aktion für EM gekoppelt an n massive Punktladungen q 1 , , q n , an Positionen r 1 , , r n , wird angegeben als

(13) S [ EIN μ ; r ich ]   =   d t   L ,

wo der Lagrange ist

(14) L   =   1 4 d 3 r   F μ v F μ v ich = 1 n ( m 0 ich c 2 γ ( v ich ) + q ich { ϕ ( r ich ) v ich EIN ( r ich ) } ) .

Die entsprechenden Euler-Lagrange-Gleichungen. sind 4 Maxwell-Gleichungen. mit Quellen (bei Variation EIN μ ) , und n (speziell relativistisch) Newtons 2. Gesetze mit Lorentzkräften (bei Variation r ich ) .

Danke @Qmechaniker! Bedeutet dies, dass wir selbst angesichts der Bewegungsgleichungen (wie Maxwell-Gleichungen für die EM-Felddynamik und Lorentz-Kraftgleichungen für die Ladungsdynamik) die Lagrange-Funktion nicht aus Bewegungsgleichungen reproduzieren können? Wenn wir beispielsweise magnetische Ladungen ungleich Null zulassen, können wir keine Potentiale verwenden, aber wir haben immer noch gültige Maxwell- und Lorentz-Kraftgleichungen (natürlich angepasst an die Existenz magnetischer Ladungen). Können wir in diesem Fall das Prinzip der kleinsten Wirkung formulieren?
Ich habe die Antwort aktualisiert. Die Frage nach den Wirkungsprinzipien für magnetische Monopole ist ein großes separates Thema und sollte als separate Frage gepostet werden. Neben der anderen Phys.SE- Frage von OP wurde in diesem Phys.SE-Beitrag und deren Links auch ein verwandtes Thema zu Potentialen in Gegenwart magnetischer Monopole gestellt.

Ich weiß nicht, ob ein anderer Ansatz möglich ist, aber dieser funktioniert nicht. Wir beginnen mit Tensor F μ v :

F μ v = μ EIN v v EIN μ

aber vergiss das 4-Potenzial und definiere es als:

F μ v = [ 0 E x / c E j / c E z / c E x / c 0 B z B j E j / c B z 0 B x E z / c B j B x 0 ]

und schreiben Sie die Lagrange-Dichte als Funktion der kartesischen Komponenten der Felder, sagen Sie:

L = L ( E x , . . , B z )

und

L = 1 μ 0 F μ v F μ v

Dann gibt man Euler-Lagrange-Gleichungen (zum Beispiel angewendet auf E x ) L E x = 0 das ist also nicht konsequent.

Wie können wir das Problem lösen? In einer anderen Lagrange-Dichte denken, etwas Neues definieren F μ v Tensor, sorgfältigere Auswahl der unabhängigen Felder?

Es ist bekannt, dass der EM-Lagrangian einen zusätzlichen Begriff enthält EIN μ J v .

Warum nicht versuchen? zum Beispiel mit einem Lagrangian der spezifischen Form "Polynom zweiter Ordnung"

(1) L ( x μ , X , μ X ) := ich , j = 1 6 ( EIN ich j X ich X j + μ = 0 3 B ich j μ μ X ich X j + μ , v = 0 3 C ich j μ v μ X ich v X j )
in X = ( E x , E j , E z , B x , B j , B z ) . (Da die klassischen Felder X ich X j = X j X ich pendeln, nur der symmetrische Teil EIN und C eine Rolle spielen, also können wir davon ausgehen, dass sie symmetrisch sind.) Unter Berücksichtigung der Antwort von Qmechanics versuchen wir nicht, alle Maxwell-Gleichungen wiederherzustellen, sondern versuchen wir zum Beispiel diejenigen mit möglichen "Quelltermen":
(2) { E = 0 B μ = ϵ 0 E t
Mit den Notationen von ( 1 ), entspricht dies dem Verschwinden von 4 linearen Formen (1 für die erste "skalare" Gleichung, 3 für die zweite Vektorgleichung; X ich i-te Komponente, nicht etwas hoch i )
(3) { a ( μ X ) := ich = 1 3 μ = 0 3 a ich μ μ X ich = ich = 1 3 μ = 0 3 δ ich μ μ X ich = ich = 1 3 ich X ich β ( μ X ) := ich = 1 6 μ = 0 3 β ich μ μ X ich = 1 μ ( j B z z B j ) ϵ 0 E x t = 1 μ ( j X 6 z X 5 ) ϵ 0 X 1 t =
Linke Seite der Euler-Lagrange-Gleichung in der Form L X ich = μ L ( μ X ich ) liest
L X ich = j = 1 6 ( 2 EIN ich j X j + μ = 0 3 B j ich μ μ X j )
und rhs (mit Einsteins Summationskonvention)
μ L ( μ X ich ) = μ [ j = 1 6 ( μ = 0 3 B ich j μ X j + 2 μ , v = 0 3 C ich j μ v v X j ) ] = j = 1 6 ( μ = 0 3 B ich j μ μ X j + 2 μ , v = 0 3 C ich j μ v μ v X j )
Man sieht das, um ( 3 ), darf die Lagrange-Funktion keine Terme enthalten EIN ich j und C ich j μ v . Dann
L X ich μ L ( μ X ich ) = j = 1 6 μ = 0 3 ( B j ich μ B ich j μ ) μ X j
(Vorsicht: B ich j μ Notation von ( 1 ), nicht das Magnetfeld...). Nur sein antisymmetrischer Teil spielt eine Rolle, also wählen wir ihn antisymmetrisch, um Eindeutigkeit zu haben. Lassen Sie uns von Hand ansetzen ich = 4 ,   0 μ 3
(4) B 4 j μ = B j 4 μ = δ j μ wenn   1 j 3 und B 4 j μ = B j 4 μ = 0 wenn   4 j 6
(also ganz explizit der folgende Beitrag   E B x B x ( E ) zum Lagrange ( 1 ))

Dies ergibt die erste Gleichung von ( 3 ) ( 1 ). Für ich = 2 ,   0 μ 3 , Lass es uns versuchen

(5) B 26 2 = B 62 2 = 1 2 μ   , B 25 3 = B 52 3 = 1 2 μ   , B 21 0 = B 12 0 = ϵ 0 2
und B 2 j μ = B j 2 μ = 0 für alle anderen ( ich = 2 , μ , j ) anders als oben.

(Expliziter Beitrag zum Lagrange:

1 2 μ ( j E j B z j B z E j ) + 1 2 μ ( z E j B j z B j E j ) + ϵ 0 2 ( t E j E x t E x E j )
)

Teilschluss: An dieser Stelle scheint es keinen Widerspruch zu geben. Für jeden fest μ ,   ( B ich j μ ) ist ein 6 × 6 antisymmetrische Matrix, also 5 unabhängige Koeffizienten u 20 insgesamt. Imposant zu erholen ( 2 - 3 ) sollte die Möglichkeiten naiv auf einen Vektorraum der Dimension reduzieren 20 4 = 16 aber die Beispiele ( 4 - 5 ) zeigen, dass man wohl zweimal überlegen sollte.

Ich bin mir ziemlich sicher, dass ich in Arxiv eine Zeitung gelesen habe, in der so etwas bereits gemacht wurde. Leider habe ich den Namen vergessen und bin mit der Suche nicht fündig geworden.
Ok, ich werde danach suchen, wenn ich Zeit habe, und diesen Kommentar durch einen Link zu diesem Artikel ersetzen

In der klassischen Elektrodynamik sind die interessierenden physikalischen Größen die Felder. Die Theorie ist bereits „ohne“ Potentiale formuliert, wenn man an die Maxwellschen Gleichungen denkt.

Die Potentiale kommen später ins Spiel, wenn man die Gleichungen vereinfachen und Lösungen zB mit Green'schen Funktionen etc. finden will. In der Quantenelektrodynamik spielen die Potentiale aber eine echte physikalische Rolle, siehe zB den Aharanov-Bohm-Effekt.

Wie geht dies mit "...aber mit [einem] Prinzip der kleinsten Aktion" um?
Danke, @RobertFilter. Ich denke jedoch, dass die Bedeutung des Aharonov-Bohm-Effekts überbewertet wird. Alle Spekulationen über die physikalische Rolle von Potentialen gelten nur, wenn man Teilchen als punktförmig betrachtet.
Die Kopplung an Materie betrifft explizit die Potentiale.
@JerrySchirmer Das stimmt, wenn wir den Interaktionsterm wie gewohnt im Lagrange schreiben μ EIN μ . Wenn Sie jedoch auf die Maxwell-Gleichungen und die Lorentz-Kraft zurückkommen, j v F μ v , die Potentiale sind in keiner Bewegungsgleichung in CED enthalten. Btw: Wer hat auf meinen Kommentar geantwortet?!
@RobertFilter: Die Maxwell-Gleichungen und die Lorentz-Kraft sind nicht Teil des Aktionsprinzips. Der Lagrange ist 1 4 F a b F a b + EIN μ j μ . Die meisten Materie-Lagrangianer werden die Eichinvarianz brechen, daher ist dies nicht trivial.
@JerrySchirmer Ja, du hast Recht, ich hatte einen Tippfehler, mein Interaktionsbegriff sollte natürlich lauten j μ EIN μ wie du sagst. Die Bewegungsgleichungen von CED sind jedoch Maxwells-Gleichungen und in ihnen gibt es keine Potentiale, was alles ist, was ich sage. Wie auch immer, reale Materiewechselwirkungen sind in CED natürlich schwer zu handhaben, wie wir zum Beispiel an der umständlichen Ableitung der Strahlungsreaktion sehen können.